lunes, 5 de junio de 2017

Revisitando la demostración de Erdös del postulado de Bertrand

Al releer recientemente el artículo donde Erdös expone su prueba del postulado de Bertrand, caí en la cuenta de que el buen Paul sí mencionó en ese trabajo que su prueba daba también una cota inferior para el número de primos en los intervalos $(n,2n]$ (donde $n \in \mathbb{N}$) y que dicha cota es prácticamente la que predice el teorema de los números primos.

Recordemos que lo que Erdös hace en su artículo es acotar inferior y superiormente los coeficientes binomiales $c_{n}:=\binom{2n}{n}$ y comparar entre sí sendas estimaciones. La estimación inferior es $$\frac{4^{n}}{2n} \leq \binom{2n}{n}$$ y la obtiene de la identidad $\binom{2n}{0} + \ldots + \binom{2n}{2n} = 2^{2n} = 4^{n}$. La estimación superior la obtiene al estudiar de una manera muy astuta la descomposición en números primos de $\binom{2n}{n}$: en efecto, del teorema fundamental de la aritmética, de la fórmula Legendre (cf. A. M. Legendre, Essai sur la théorie des nombres. Seconde édition, 1808, págs. 8-10; de acuerdo con W. Narkiewicz, la atribución a de Polignac (1826-1863) y/o a Chebyshev (1821-1894) de este resultado es incorrecta) y de la desigualdad de Erdös-Kalmár se llega a que \begin{eqnarray*} \binom{2n}{n} &\leq & \prod_{p \leq \sqrt{2n}} p^{\alpha_{p}(c_{n})} \prod_{\sqrt{2n} < p \leq \frac{2}{3}n} p \prod_{n < p \leq 2n} p\\ &\leq& \prod_{p \leq \sqrt{2n}} (2n) \cdot 4^{\frac{2}{3}n} \cdot \prod_{n < p \leq 2n} p\\ &\leq& (2n)^{\sqrt{2n}} \cdot 4^{\frac{2}{3}n} \cdot \prod_{n < p \leq 2n} p \end{eqnarray*} para cada número natural $n \geq 3$. La conexión con el estudio de los números primos en $(n,2n]$ se acaba de hacer más que patente en este momento, ¿cierto?

De ambas estimaciones se desprende que si $n\geq 3$ entonces \begin{eqnarray*} 4^{\frac{n}{3}} \leq (2n)^{1+\sqrt{2n}} \prod_{n < p \leq 2n} p. \end{eqnarray*} De esto y de la desigualdad $$ (2n)^{1+\sqrt{2n}} < 2^{\frac{n}{2}},$$ la cual es válida para todo número natural $n$ suficientemente grande (lo que en este caso quiere decir, en números redondos, siempre que $n > 22 \, 620$), se obtiene que $$ \prod_{n < p \leq 2n} p > 2^{\frac{n}{6}}$$ si $n$ es suficientemente grande: ergo, para cada $n$ así de grande se cumple que $$(2n)^{\pi(2n)-\pi(n)} > 2^{\frac{n}{6}},$$ o equivalentemente que \begin{eqnarray*} \pi(2n)-\pi(n) > \frac{\log 2}{6} \cdot \frac{n}{\log 2n}. \end{eqnarray*} En resumidas cuentas: ¡la formulación clásica del postulado de Bertrand es sumamente conservadora!

1 comentario:

Nilton Raúl Olivares Ramírez dijo...

He descubierto una demostración elemental, es decir, muy básica o sencilla del Postulado de Bertrand, espero publicarla pronto. Saludos desde el Perú.